Download as pdf or txt
Download as pdf or txt
You are on page 1of 55

Government & Administration CSE PYQs [1995 -

2020]
Note : CSE 2021 [all] Qs are included in Constitution & Political System (Polity) PYQs [1995 -2021]

Set 1
Question 1: Survey of India is under the ministry of: [2003]
(a) Defence
(b) Environment and Forests
(c) Home Affairs
(d) Science and Technology
Correct Answer is Option (d)
Survey of India, The National Survey and Mapping Organization of the country under the
Department of Science and Technology, is the oldest scientific department of the Govt. of India. It
was set up in 1767.

Question 2: With reference to Indian Parliament, which one of the following is not
correct? [2004]
(a) The Appropriation Bill must be passed by both the Houses of Parliament before it can be
enacted into law
(b) No money shall be withdrawn from the Consolidated Fund of India except under the
appropriation made by the Appropriation Act
(c) Finance Bill is required for proposing new taxes but no additional Bill/Act is required for
making changes in the rates of taxes which are already under operation.
(d) No Money Bill can be introduced except on the recommendation of the President
Correct Answer is Option (a)
Appropriation Bill is a money bill. In case of money bill, RS has only recommendatory power and
need not to be passed by RS.

Question 3: Consider the following tasks: [2004]


1. Superintendence, direction and conduct of free and fair elections
2. Preparation of electoral rolls for all elections to the Parliament, state Legislatures and the
Office of the President and the Vice-President
3. Giving recognition to political, parties and allotting election symbols to political parties
and individuals contesting the election.
4. Proclamation of final verdict in case of election disputes
Which of the above are the functions of the Election Commission of India?
Telegram : https://t.me/prelimbits || Personal Contact : https://t.me/EN123upsc
(a) 1, 2 and 3
(b) 2, 3 and 4
(c) 1 and 3
(d) 1, 2 and 4
Correct Answer is Option (a)
The High Court (and not the Election Commission) is the final authority to give a final verdict in
case of election disputes. In the alternative special election benches may be constituted in high
courts and earmarked exclusively for the disposal of election petitions and disputes.

Question 4: Consider the following statements: [2004]


1. The Speaker of Lok Sabha has the power to adjourn the House sine die but, on
prorogation, it is only the President who can summon the House
2. Unless sooner dissolved or there is an extension of the term, there is an automatic
dissolution of the Lok Sabha by efflux of time, at the end of the period of five years, even if
no formal order of dissolution is issued by the President
3. The Speaker of Lok Sabha continues in office even after the dissolution of the House and
until immediately before the first meeting of the House
Which of the statements given above are correct?
(a) 1 and 2
(b) 2 and 3 .
(c) 1 and 3
(d) 1, 2 and 3
Correct Answer is Option (d)
Lok Sabha speaker is the presiding officer or head of Lok Sabha. He is the guardian of power and
privileges of members and committees of Lok Sabha. The Speaker of the Lok Sabha presides over
the joint sitting of the two Houses. "Adjournment" is a postponement of the sitting or proceedings
of the House from one time to another specified for the reassembling of the House. During the
course of a session, the Lok sabha may be adjourned from day to day or for more than a day. It
may also be adjourned sine die which means the termination of a sitting of the House without any
definite date being fixed for its next sitting.

Question 5: Which one of the following statements is not correct? [2004]


(a) In the Lok Sabha, a no-confidence motion has to set out the grounds on which it is based
(b) In the case of a no-confidence motion in Lok Sabha, no conditions of admissibility have
been laid down in the Rules
(c) A motion of no-confidence once admitted, has to be taken up within ten days of the
leave being granted
(d) Rajya Sabha is not empowered to entertain a motion of no-confidence
Telegram : https://t.me/prelimbits || Personal Contact : https://t.me/EN123upsc
Correct Answer is Option (a)
In case of a No-confidence motion, there is no need to set out the grounds on which it is based.
No-Confidence motion is introduced only in the Lok Sabha by the opposition and needs a support
of not less than 50 members of LS for its introduction. Rule 198 of the Lok Sabha specifies the
procedure for a motion of no-confidence. Any member may give a written notice; the speaker shall
read the motion of no-confidence in the House and ask all those persons to rise who favours that
the motion be taken up. If there are 50 MPs in favour, the speaker allots a date for discussing the
motion

Question 6: The resolution for removing the Vice-President of India can be moved in
the: [2004]
(a) Lok Sabha alone
(b) Either House of Parliament
(c) Joint Sitting of Parliament
(d) Rajya Sabha alone
Correct Answer is Option (d)
Article 67(b) in the Constitution of India states, a Vice President may be removed from his office by
a resolution of the council of States passed by a majority of all the then members of the council
and agreed to by the House of the People; but no resolution for the purpose of this clause shall be
moved unless at least fourteen days notice has been given of the intention to move the resolution.

Question 7: With reference to the Constitution of India, which one of the following pairs is
not correctly matched? [2004]
(a) Forests : Concurrent List
(b) Stock Exchange : Concurrent List
(c) Post Office Savings Bank : Union List
(d) Public Health : State List
Correct Answer is Option (b)
Stock Exchanges are listed in the Seventh Schedule (Article 246) List I-Union List, item no. 90 that
reads, taxes other than stamp duties on transactions in stock exchanges and futures markets.
Forests-Concurrent List, 17-A, Post Office Savings Bank -Union List-3, Public health and sanitation;
hospitals and dispensaries - State List -6.

Question 8: Consider the following statements: [2004]


1. The highest deciding body for planning in India is the Planning Commission of India

Telegram : https://t.me/prelimbits || Personal Contact : https://t.me/EN123upsc


2. The Secretary of the Planning Commission of India is also the Secretary of National
Development Council
3. The Constitution includes economic and social planning in the Concurrent List in the
Seventh Schedule of the Constitution of India
Which of the statements given above is/are correct?
(a) 1 and 2
(b) 2 and 3
(c) 2 only
(d) 3 only
Correct Answer is Option (b)
The highest decision making body for planning in India is the Parliament, National Development
Council is at second position and Planning Commission at third position. Economic and social
planning is placed under Entry 20 in the Concurrent list.

Question 9: Consider the following events: [2004]


1. Fourth general elections in India
2. Formation of Haryana state
3. Mysore named as Karnataka state
4. Meghalaya and Tripura become full states
Which one of the following is the correct chronological order of the above?
(a) 2, 1, 4, 3
(b) 4, 3, 2, 1
(c) 2, 3, 4, 1
(d) 4, 1, 2, 3
Correct Answer is Option (a)
Haryana was founded in 1966 when the former state of Punjab was divided into Haryana and the
modern Punjab. The 1967 general elections or the 4th Lok Sabha was held in February. Tripura,
Manipur and Meghalaya became fullfledged states on Jan 21, 1972. Mysore state was renamed
Karnataka in 1997.

Question 10: Under which one of the Ministries of the Government of India does the Food
and Nutrition Board work? [2005]
(a) Ministry of Agriculture
(b) Ministry of Health and Family Welfare
(c) Ministry of Human Resource Development
(d) Ministry of Rural Development
Correct Answer is Option (*)

Telegram : https://t.me/prelimbits || Personal Contact : https://t.me/EN123upsc


Food and Nutrition Board works under Ministry of Women and Child Development. It is a technical
support wing under Child Development Bureau of the Ministry. None of the given options is
correct.

Question 11: Consider the following statements: [2006]


1. The Rajya Sabha alone has the power to declare that it would be in national interest for
the Parliament to legislate with respect to a matter in the State List.
2. Resolutions approving the proclamation of Emergency are passed only by the Lok Sabha.
Which of the statement(s) given above is/are correct?
(a) 1 only
(b) 2 only
(c) Both 1 and 2
(d) Neither 1 nor 2
Correct Answer is Option (a)
Statement 1 is correct as per provisions under Article 249. Statement 2 is incorrect as resolutions
approving the proclamation of Emergency are passed by both Houses of Parliament (not only LS).

Question 12:
Assertion (A): The Council of Ministers in the Union of India is collectively responsible both
to the Lok Sabha and Rajya Sabha. [2007]
Reason (R): The Members of both the Lok Sabha and the Rajya Sabha are eligible to be the
Ministers of the Union Government.
(a) Both A are R are true and R is the correct explanation of A
(b) Both A and R are true but R is not a correct explanation of A
(c) A is true but R is false
(d) A is false but R is true
Correct Answer is Option (d)
Assertion is false, because the Council of Ministers in the Union of India is collectively responsible
to the Lok Sabha only.

Question 13: Consider the following statements: [2007]


1. Jawaharlal Nehru was in his fourth term as the Prime Minister of India at the time of his
death.
2. Jawaharlal Nehru represented Rae Bareilly constituency as a Member of Parliament.
3. The first non-Congress Prime Minister of India assumed the office in the year 1977.

Telegram : https://t.me/prelimbits || Personal Contact : https://t.me/EN123upsc


Which of the statements given above is/are correct?
(a) 1 and 2
(b) 3 only
(c) 1 only
(d) 1 and 3
Correct Answer is Option (d)
Terms of Jawahar Lal Nehru (1947-52; 1952-57; 1957- 62; 1962-64). He died in 1964 during his 4th
Prime-ministerial term. He represented Phulpur constituency in UP. The first non-congress PM of
India was Morarji Desai (Janta Party) from 1977-1979.

Question 14: Who was the Speaker of the First Lok Sabha? [2007]
(a) Hukum Singh
(b) G.V. Mavalankar
(c) K. M. Munshi
(d) U.N. Dhebar
Correct Answer is Option (b)
GV Mavalankar (1952-56), Hukum Singh (1962-67); K.M. Munshi and U.N. Dhebar were never the
Speakers of the Lok Sabha.

Question 15: Consider the following statements in respect of financial emergency under
Article 360 of the Constitution of India: [2007]
1. A proclamation of financial emergency issued shall cease to operate at the expiration of
two months, unless before the expiration of that period it has been approved by the
resolutions of both Houses of Parliament.
2. If any proclamation of financial emergency is in operation, it is competent for the
President of India to issue directions for the reduction of salaries and allowances of all or
any class of persons serving in connection with the affairs of the Union but excluding the
Judges of Supreme Court and the High Courts.
Which of the statements given above is/are correct?
(a) 1 only
(b) 2 only
(c) Both 1 and 2
(d) Neither 1 nor 2
Correct Answer is Option (a)
Statement 1 is correct as under Article 360, any Proclamation of Financial Emergency issued shall
cease to operate at the expiration of two months, unless before the expiration of that period it has
been approved by the resolutions Passed by both Houses of Parliament. If approved by both

Telegram : https://t.me/prelimbits || Personal Contact : https://t.me/EN123upsc


Houses, then it operates for 6 months. During the proclamation of financial emergency. President
of India can issue directions for the reduction of salaries and allowances of all or any class of
persons serving in connection with the affairs of the union including the judges of supreme court
and the High warts.

Question 16: Consider the following statements: [2007]


1. The Chairman of the Committee on Public Accounts is appointed by the Speaker of the
Lok Sabha.
2. The Committee on Public Accounts comprises Members of Lok Sabha, Members of Rajya
Sabha and few eminent persons of industry and trade.
Which of the statements given above is/are correct?
(a) 1 only
(b) 2 only
(c) Both 1 and 2
(d) Neither 1 nor 2
Correct Answer is Option (a)
Statement 2 is incorrect as Public Accounts Committee consists of 22 members: 15 from Lok Sabha
and 7 from Rajya Sabha.

Question 17: Who among the following have held the office of the Vice- President of
India? [2008]
1. Mohammad Hidayatullah
2. Fakhruddin Ali Ahmed
3. Neelam Sanjiva Reddy
4. Shankar Dayal Sharma
Select the correct answer using the code given below:
(a) 1, 2, 3 and 4
(b) 1 and 4 only
(c) 2 and 3 only
(d) 3 and 4 only
Correct Answer is Option (b)
Mohd. Hidayatullah (1979-84); Shankar Dayal Sharma (1987-92)

Question 18: Match List-I with List-II and select the correct answer using the code given
below the Lists: [2008]

Telegram : https://t.me/prelimbits || Personal Contact : https://t.me/EN123upsc


(a) A-1; B-4; C-2; D-3
(b) A-2; B-3; C-1; D-4
(c) A-1; B-3; C-2; D-4
(d) A-2; B-4; C-1; D-3
Correct Answer is Option (b)
Ashok Desai was Attorney General of India (1996-98); Nagendra Singh was President of
International Court of Justice (1985-88); AN Ray was CJI (1973-77); RK Trivedi was Chief Election
Commissioner (1982-85)

Question 19: Consider the following statements: [2008]


The Constitution of India provides that:
1. the Legislative Assembly of each State shall consist of not more than 450 members chosen
by direct election from territorial constituencies in the State
2. a person shall not be qualified to be chosen to fill a seat in the Legislative Assembly of a
State if he/she is less than 25 years of age
Which of the statements given above is/are correct?
(a) 1 only
(b) 2 only
(c) Both 1 and 2
(d) Neither 1 nor 2
Correct Answer is Option (b)
Statement 2 is correct as per provisions given under Article 173. Statement 1 is incorrect as
according to article 170, the legislative assembly of each state shall consist of not more than 500
and not less than 60 members chosen by direct election from territorial constituencies in the state.

Question 20: With reference to the Union Government, consider the following
statements: [2009]

Telegram : https://t.me/prelimbits || Personal Contact : https://t.me/EN123upsc


1. The Constitution of India provides that all Cabinet Ministers shall be compulsorily be a
sitting members of Lok Sabha only.
2. The Union Cabinet Secretariat operates under the direction of the Ministry of
Parliamentary Affairs.
Which of the statements given above is/are correct?
(a) 1 only
(b) 2 only
(c) Both 1 and 2
(d) Neither 1 nor 2
Correct Answer is Option (d)
Statement 1 is incorrect as members of Rajya Sabha can become cabinet ministers. Statement 2 is
incorrect as Cabinet secretariat is under the direct charge of the PM.

Question 21: With reference to Union Government, consider the following


statements: [2009]
1. The Ministries and Departments of the Government of India are created by the Prime
Minister on the advice of the Cabinet Secretary.
2. Each of the ministries is assigned to a Minister by the President of India on the advice of
the Prime Minister.
Which of the statements given above is/are correct?
(a) 1 only
(b) 2 only
(c) Both 1 and 2
(d) Neither 1 nor 2
Correct Answer is Option (b)
According to Article 70 the Prime Minister shall be appointed by the President and the other
Ministers shall be appointed by the President on the advice of the Prime Minister. The government
of India (Allocation of Business) Rules, 1961 is made by the President of India under Article 77 of
the constitution for the allocation of business of the government of India.
The Ministries/ Departments of the government of India are created by the president on the advice
of the prime Minister under these Rules.

Question 22: Consider the following statements: [2009]


1. The Advocate General of a State in India is appointed by the President of India upon the
recommendation of the Governor of the concerned State.
2. As provided in Civil Procedure Code, High Courts have original, appellate and advisory
jurisdiction at the State level.

Telegram : https://t.me/prelimbits || Personal Contact : https://t.me/EN123upsc


Which of the statements given above is/are correct?
(a) 1 only
(b) 2 only
(c) Both 1 and 2
(d) Neither 1 nor 2
Correct Answer is Option (b)
Statement 1 is incorrect as Advocate General of the state is appointed by the governor of the
State. Statement 2 is correct. The original civil jurisdiction of the High Courts
has not altogether been abolised but retained in respect of actions of higher value.
The Original Criminal Jurisdiction of High Courts has, however, been completely taken away by the
criminal procedure code, 1973.

Question 23: Consider the following statements: [2009]


1. Central Administrative Tribunal (CAT) was set up during the Prime Ministership of Lal
Bahadur Shastri.
2. The Members of CAT are drawn from both judicial and administrative streams.
Which of the statements given above is/are correct?
(a) 1 only
(b) 2 only
(c) Both 1 and 2
(d) Neither 1 nor 2
Correct Answer is Option (b)
Statement 1 is incorrect as CAT was set up in 1985 during the prime ministership of Rajiv Gandhi.

Question 24: With reference to Union Government, consider the following


statements: [2009]
1. Number of Ministries at the Centre on 15th August 1947 was 18.
2. Number of Ministries at the Centre at present is 36.
Which of the statements given above is/are correct?
(a) 1 only
(b) 2 only
(c) Both 1 and 2
(d) Neither 1 nor 2
Correct Answer is Option (a)

Telegram : https://t.me/prelimbits || Personal Contact : https://t.me/EN123upsc


The number of ministeries at the Centre can vary based on factors such as volume of work,
importance attached to different sectors, changes of orientation of policy etc. On 15 Aug, 1947,
the number of ministries at the centre was 18.

Question 25: With reference to the Consumer Disputes Redressal at district level in India,
which one of the following statements is not correct ? [2010]
(a) A State Government can establish more than one District Forum in a district if it deems
fit.
(b) One of the members of the District Forum shall be a woman
(c) The District Forum entertains the complaints where the value of goods or services does
not exceed rupees fifty lakhs.
(d) A complaint in relation to any goods sold or any service provided may be filed with a
District Forum by the State Government as a representative of the interests
of the consumers in general.
Correct Answer is Option (c)
The District Forum entertains the complaints where the value of goods or services does not exceed
rupees twenty lakhs.

Question 26: When the annual Union Budget is not passed by the Lok Sabha ? [2011 - I]
(a) The Budget is modified and presented again
(b) The Budget is referred to the Rajya Sabha for suggestions
(c) The Union Finance Minister is asked to resign
(d) The Prime Minister submits the resignation of Council of Ministers.
Correct Answer is Option (d)
If annual union budget is not passed by the LOK SABHA, it is tantamount to no confidence motion.
So the govt submits the resignation of his council of ministers.

Question 27: The authorization for the withdrawal of funds from the Consolidated Fund of
India must come from [2011 - I]
(a) The President of India
(b) The Parliament of India
(c) The Prime Minister of India
(d) The Union Finance Minister
Correct Answer is Option (b)
Parliament shall have power to authorise by law the withdrawal of moneys from the Consolidated
Fund of India for the purposes for which the said grants are made.

Telegram : https://t.me/prelimbits || Personal Contact : https://t.me/EN123upsc


Question 28: In the Parliament of India, the purpose of an adjournment
motion is [2012 - I]
(a) to allow a discussion on a definite matter of urgent public importance
(b) to let opposition members collect information from the ministers
(c) to allow a reduction of specific amount in demand for grant
(d) to postpone the proceedings to check the inappropriate or violent behaviour on the part
of some members
Correct Answer is Option (a)
Adjournment motion:
1. It is introduced in the Parliament to draw attention of the house to a definite matter of
urgent public importance.
2. It is regarded as an extraordinary device, because it interrupts the normal business of the
house.
3. It involves an element of censure against the government and Rajya Sabha cannot
introduce Adjournment Motion.
4. The discussion on adjournment motion should last for not less than two hours and thirty
minutes.

Question 29: Consider the following statements: [2012 - I]


1. Union Territories are not represented in the Rajya Sabha.
2. It is within the purview of the Chief Election Commissioner to adjudicate the election
disputes.
3. According to the Constitution of India, the Parliament consists of the Lok Sabha and the
Rajya Sabha only.
Which of the statements given above is/are correct?
(a) 1 only
(b) 2 and 3
(c) 1 and 3
(d) None
Correct Answer is Option (d)
None of the above statements are correct
1. Union Territories (Delhi and Pondicherry) are represented in the Rajya Sabha.
2. It is not within the purview of the Chief Election Commissioner to adjudicate election
disputes. It is the Supreme Court and High Court which look into the disputes.

Telegram : https://t.me/prelimbits || Personal Contact : https://t.me/EN123upsc


3. According to the Constitution of India, the Parliament consists of the Lok Sabha, the Rajya
Sabha and the President of India.

Question 30: Regarding the office of the Lok Sabha Speaker, consider the following
statements: [2012 - I]
1. He/She holds the office during the pleasure of the President.
2. He/She need not be a member of the House at the time of his/her election but has to
become a member of the House within six months from the date of his/her election.
3. If he/she intends to resign, the letter of his/her resignation has to be addressed to the
Deputy Speaker.
Which of the statements given above is /are correct?
(a) 1 and 2 only
(b) 3 only
(c) 1, 2 and 3
(d) None
Correct Answer is Option (b)
If the Lok Sabha Speaker wants to resign, the letter of his / her resignation has to be addressed to
the Deputy Speaker. The Speaker is elected by the Lok-Sabha from amongst its Members. Usually
the Speaker remains in office during the life of the Lok-Sabha.

Question 31: Which of the following can be said to be essentially the parts of ‘Inclusive
Governance’? [2012 - I]
1. Permitting the Non-Banking Financial Companies to do banking
2. Establishing effective District Planning Committees in all the districts
3. Increasing the government spending on public health
4. Strengthening the Mid-day Meal Scheme
Select the correct answer using the codes given below :
(a) 1 and 2 only
(b) 3 and 4 only
(c) 2, 3 and 4 only
(d) 1, 2, 3 and 4
Correct Answer is Option (c)
Inclusive governance Means that the benefit of governance Should reach the down trodden and to
the last corner of the country.
Provisions given in option 2, 3 and 4 leads to the inclusive governance.
Permitting the Non-Banking Financial companies to do banking is not directly linked to the
inclusive governance.
Telegram : https://t.me/prelimbits || Personal Contact : https://t.me/EN123upsc
Question 32: In India, other than ensuring that public funds are used efficiently and for
intended purpose, what is the importance of the office of the Comptroller and Auditor
General (CAG)? [2012 - I]
1. CAG exercises exchequer control on behalf of the Parliament when the President of India
declares national emergency/financial emergency.
2. CAG reports on the execution of projects or programmes by the ministries are discussed
by the Public Accounts Committee.
3. Information from CAG reports can be used by investigating agencies to frame charges
against those who have violated the law while managing public finances.
4. While dealing with the audit and accounting of government companies, CAG has certain
judicial powers for prosecuting those who violate the law.
Which of the statements given above is/are correct?
(a) 1, 3 and 4 only
(b) 2 only
(c) 2 and 3 only
(d) 1, 2, 3 and 4
Correct Answer is Option (c)
1. Only 2nd and 3rd are correct statements.
2. CAG reports on execution of projects or programmes by the ministries are discussed by the
Public Accounts Committee.
3. Information from CAG reports can be used by investigating agencies to press charges
against those who have violated the law while managing public finance.

Question 33: The Prime Minister of India, at the time of his/her appointment [2012 - I]
(a) need not necessarily be a member of one of the Houses of the Parliament but must
become a member of one of the Houses within six months
(b) need not necessarily be a member of one of the Houses of the Parliament but must
become a member of the Lok Sabha within six months
(c) must be a member of one of the Houses of the Parliament
(d) must be a member of the Lok Sabha
Correct Answer is Option (a)
Article 75(I) of the Indian Constitution provides that the Prime Minister shall be appointed by the
President. The Constitution permits a person to be appointed PM without his\her being a member
of either House of the Parliament at the time of appointment. However he/she has to become a

Telegram : https://t.me/prelimbits || Personal Contact : https://t.me/EN123upsc


Member of either house of parliament with in Six Months from the date of his/her appointment as
prime-minister.

Question 34: A deadlock between the Lok Sabha and the Rajya Sabha calls for a joint sitting
of the Parliament during the passage of [2012 - I]
1. Ordinary Legislation
2. Money Bill
3. Constitution Amendment Bill
Select the correct answer using the codes given below :
(a) 1 only
(b) 2 and 3 only
(c) 1 and 3 only
(d) 1, 2 and 3
Correct Answer is Option (a)
Joint sitting is an extra-ordinary Machinery provided by the constitution to resolve a deadlock
between the two houses over the passage of a bill. The provision of joint sitting is applicable to
ordinary bills of financial bills only and not to Money bills or constitutional amendment bills.

Question 35: What will follow if a Money Bill is substantially amended by the Rajya
Sabha? [2013 - I]
(a) The Lok Sabha may still proceed with the Bill, accepting or not accepting the
recommendations of the Rajya Sabha
(b) The Lok Sabha cannot consider the bill further
(c) The Lok Sabha may send the Bill to the Rajya Sabha for reconsideration
(d) The President may call a joint sitting for passing the Bill
Correct Answer is Option (a)
When a money bill returns to the Loksabha with amendments made by the Rajyasabha, it is open
to Loksabha to accept or to reject any or all of the recommendations. When the Loksabha chooses
to accept or decline the money bill with or without the recommendation, the money bill is deemed
passed in both houses.

Question 36: Which one of the following statements is correct? [2013 - I]


(a) In India, the same person cannot be appointed as Governor for two or more States at the
same time
(b) The Judges of the High Court of the States in India are appointed by the Governor of the
State just as the Judges of Supreme Court are appointed by the President
(c) No procedure has been laid down in the Constitution of India for the removal of a

Telegram : https://t.me/prelimbits || Personal Contact : https://t.me/EN123upsc


Governor from his/her post
(d) In the case of a Union Territory having a legislative setup, the Chief Minister is appointed
by the Lt. Governor on the basis of majority support.
Correct Answer is Option (c)
No procedure has been laid down in the constitution of India for the removal of a governor from
his/her post.

Question 37: Consider the following statements : Attorney General of India can [2013 - I]
1. take part in the proceedings of the Lok Sabha
2. be a member of a committee of the Lok Sabha
3. speak in the Lok Sabha
4. vote in the Lok Sabha
Which of the statements given above is/are correct?
(a) 1 only
(b) 2 and 4
(c) 1, 2 and 3
(d) 1 and 3 only
Correct Answer is Option (c)
The Attorney General of India has a post parallel to any minister in Parliament. He can take part in
the proceedings of either house. He can be a member of any committee of Parliament. He has the
right to speak in the Parliament but he has no right to vote.

Question 38: The Parliament can make any law for whole or any part of India for
implementing international treaties [2013 - I]
(a) with the consent of all the States
(b) with the consent of the majority of States
(c) with the consent of the States concerned
(d) without the consent of any State
Correct Answer is Option (d)
Parliament has exclusive power to make law with respect to any of the matters enumerated with
the Union List. According to entry no 14 in the Union List it reads- „entering into treaties and
agreements with foreign contries and implementing of treaties, agreement and convention with
foreign countries‟.

Question 39: In the context of India, which of the following principles is/are implied
institutionally in the parliamentary government? [2013 - I]
1. Members of the Cabinet are Members of the Parliament.
Telegram : https://t.me/prelimbits || Personal Contact : https://t.me/EN123upsc
2. Ministers hold the office till they enjoy confidence in the Parliament.
3. Cabinet is headed by the Head of the State.
Select the correct answer using the codes given below.
(a) 1 and 2 only
(b) 3 only
(c) 2 and 3 only
(d) 1, 2 and 3
Correct Answer is Option (a)
Minister/ministers can be removed by issuing no confidence motion in the parliament. All cabinet
members are mandated by the constitution to be the member of either house of the parliament of
India. The cabinet is headed by the prime minister and is advised by the cabinet secretary who also
acts as the head of Indian Administrative service.

Question 40: Consider the following statements: [2013 - I]


1. The Council of Ministers in the Centre shall be collectively responsible to the Parliament.
2. The Union Ministers shall hold the office during the pleasure of the President of India.
3. The Prime Minister shall communicate to the President about the proposals for
legislation.
Which of the statements given above is/are correct?
(a) 1 only
(b) 2 and 3 only
(c) 1 and 3 only
(d) 1, 2 and 3
Correct Answer is Option (b)
Article – 75 (4): The ministers Shall hold office during the pleasure of the president.
Article – 75 (5): The council of ministers shall be collectively responsible to the Lok-Sabha.
Article–78 (2): Prime minister shall furnish such information relating to the administration of the
affairs of the union and proposals for legislation on the president may call far.

Set 2
Question 1: Survey of India is under the ministry of: [2003]
(a) Defence
(b) Environment and Forests
(c) Home Affairs
(d) Science and Technology
Correct Answer is Option (d)
Telegram : https://t.me/prelimbits || Personal Contact : https://t.me/EN123upsc
Survey of India, The National Survey and Mapping Organization of the country under the
Department of Science and Technology, is the oldest scientific department of the Govt. of India. It
was set up in 1767.

Question 2: With reference to Indian Parliament, which one of the following is not
correct? [2004]
(a) The Appropriation Bill must be passed by both the Houses of Parliament before it can be
enacted into law
(b) No money shall be withdrawn from the Consolidated Fund of India except under the
appropriation made by the Appropriation Act
(c) Finance Bill is required for proposing new taxes but no additional Bill/Act is required for
making changes in the rates of taxes which are already under operation.
(d) No Money Bill can be introduced except on the recommendation of the President
Correct Answer is Option (a)
Appropriation Bill is a money bill. In case of money bill, RS has only recommendatory power and
need not to be passed by RS.

Question 3: Consider the following tasks: [2004]


1. Superintendence, direction and conduct of free and fair elections
2. Preparation of electoral rolls for all elections to the Parliament, state Legislatures and the
Office of the President and the Vice-President
3. Giving recognition to political, parties and allotting election symbols to political parties
and individuals contesting the election.
4. Proclamation of final verdict in case of election disputes
Which of the above are the functions of the Election Commission of India?
(a) 1, 2 and 3
(b) 2, 3 and 4
(c) 1 and 3
(d) 1, 2 and 4
Correct Answer is Option (a)
The High Court (and not the Election Commission) is the final authority to give a final verdict in
case of election disputes. In the alternative special election benches may be constituted in high
courts and earmarked exclusively for the disposal of election petitions and disputes.

Question 4: Consider the following statements: [2004]


1. The Speaker of Lok Sabha has the power to adjourn the House sine die but, on
prorogation, it is only the President who can summon the House

Telegram : https://t.me/prelimbits || Personal Contact : https://t.me/EN123upsc


2. Unless sooner dissolved or there is an extension of the term, there is an automatic
dissolution of the Lok Sabha by efflux of time, at the end of the period of five years, even if
no formal order of dissolution is issued by the President
3. The Speaker of Lok Sabha continues in office even after the dissolution of the House and
until immediately before the first meeting of the House
Which of the statements given above are correct?
(a) 1 and 2
(b) 2 and 3 .
(c) 1 and 3
(d) 1, 2 and 3
Correct Answer is Option (d)
Lok Sabha speaker is the presiding officer or head of Lok Sabha. He is the guardian of power and
privileges of members and committees of Lok Sabha. The Speaker of the Lok Sabha presides over
the joint sitting of the two Houses. "Adjournment" is a postponement of the sitting or proceedings
of the House from one time to another specified for the reassembling of the House. During the
course of a session, the Lok sabha may be adjourned from day to day or for more than a day. It
may also be adjourned sine die which means the termination of a sitting of the House without any
definite date being fixed for its next sitting.

Question 5: Which one of the following statements is not correct? [2004]


(a) In the Lok Sabha, a no-confidence motion has to set out the grounds on which it is based
(b) In the case of a no-confidence motion in Lok Sabha, no conditions of admissibility have
been laid down in the Rules
(c) A motion of no-confidence once admitted, has to be taken up within ten days of the
leave being granted
(d) Rajya Sabha is not empowered to entertain a motion of no-confidence
Correct Answer is Option (a)
In case of a No-confidence motion, there is no need to set out the grounds on which it is based.
No-Confidence motion is introduced only in the Lok Sabha by the opposition and needs a support
of not less than 50 members of LS for its introduction. Rule 198 of the Lok Sabha specifies the
procedure for a motion of no-confidence. Any member may give a written notice; the speaker shall
read the motion of no-confidence in the House and ask all those persons to rise who favours that
the motion be taken up. If there are 50 MPs in favour, the speaker allots a date for discussing the
motion

Question 6: The resolution for removing the Vice-President of India can be moved in
the: [2004]
(a) Lok Sabha alone
Telegram : https://t.me/prelimbits || Personal Contact : https://t.me/EN123upsc
(b) Either House of Parliament
(c) Joint Sitting of Parliament
(d) Rajya Sabha alone
Correct Answer is Option (d)
Article 67(b) in the Constitution of India states, a Vice President may be removed from his office by
a resolution of the council of States passed by a majority of all the then members of the council
and agreed to by the House of the People; but no resolution for the purpose of this clause shall be
moved unless at least fourteen days notice has been given of the intention to move the resolution.

Question 7: With reference to the Constitution of India, which one of the following pairs is
not correctly matched? [2004]
(a) Forests : Concurrent List
(b) Stock Exchange : Concurrent List
(c) Post Office Savings Bank : Union List
(d) Public Health : State List
Correct Answer is Option (b)
Stock Exchanges are listed in the Seventh Schedule (Article 246) List I-Union List, item no. 90 that
reads, taxes other than stamp duties on transactions in stock exchanges and futures markets.
Forests-Concurrent List, 17-A, Post Office Savings Bank -Union List-3, Public health and sanitation;
hospitals and dispensaries - State List -6.

Question 8: Consider the following statements: [2004]


1. The highest deciding body for planning in India is the Planning Commission of India
2. The Secretary of the Planning Commission of India is also the Secretary of National
Development Council
3. The Constitution includes economic and social planning in the Concurrent List in the
Seventh Schedule of the Constitution of India
Which of the statements given above is/are correct?
(a) 1 and 2
(b) 2 and 3
(c) 2 only
(d) 3 only
Correct Answer is Option (b)
The highest decision making body for planning in India is the Parliament, National Development
Council is at second position and Planning Commission at third position. Economic and social
planning is placed under Entry 20 in the Concurrent list.

Telegram : https://t.me/prelimbits || Personal Contact : https://t.me/EN123upsc


Question 9: Consider the following events: [2004]
1. Fourth general elections in India
2. Formation of Haryana state
3. Mysore named as Karnataka state
4. Meghalaya and Tripura become full states
Which one of the following is the correct chronological order of the above?
(a) 2, 1, 4, 3
(b) 4, 3, 2, 1
(c) 2, 3, 4, 1
(d) 4, 1, 2, 3
Correct Answer is Option (a)
Haryana was founded in 1966 when the former state of Punjab was divided into Haryana and the
modern Punjab. The 1967 general elections or the 4th Lok Sabha was held in February. Tripura,
Manipur and Meghalaya became fullfledged states on Jan 21, 1972. Mysore state was renamed
Karnataka in 1997.

Question 10: Under which one of the Ministries of the Government of India does the Food
and Nutrition Board work? [2005]
(a) Ministry of Agriculture
(b) Ministry of Health and Family Welfare
(c) Ministry of Human Resource Development
(d) Ministry of Rural Development
Correct Answer is Option (*)
Food and Nutrition Board works under Ministry of Women and Child Development. It is a technical
support wing under Child Development Bureau of the Ministry. None of the given options is
correct.

Question 11: Consider the following statements: [2006]


1. The Rajya Sabha alone has the power to declare that it would be in national interest for
the Parliament to legislate with respect to a matter in the State List.
2. Resolutions approving the proclamation of Emergency are passed only by the Lok Sabha.
Which of the statement(s) given above is/are correct?
(a) 1 only
(b) 2 only
(c) Both 1 and 2
(d) Neither 1 nor 2

Telegram : https://t.me/prelimbits || Personal Contact : https://t.me/EN123upsc


Correct Answer is Option (a)
Statement 1 is correct as per provisions under Article 249. Statement 2 is incorrect as resolutions
approving the proclamation of Emergency are passed by both Houses of Parliament (not only LS).

Question 12:
Assertion (A): The Council of Ministers in the Union of India is collectively responsible both
to the Lok Sabha and Rajya Sabha. [2007]
Reason (R): The Members of both the Lok Sabha and the Rajya Sabha are eligible to be the
Ministers of the Union Government.
(a) Both A are R are true and R is the correct explanation of A
(b) Both A and R are true but R is not a correct explanation of A
(c) A is true but R is false
(d) A is false but R is true
Correct Answer is Option (d)
Assertion is false, because the Council of Ministers in the Union of India is collectively responsible
to the Lok Sabha only.

Question 13: Consider the following statements: [2007]


1. Jawaharlal Nehru was in his fourth term as the Prime Minister of India at the time of his
death.
2. Jawaharlal Nehru represented Rae Bareilly constituency as a Member of Parliament.
3. The first non-Congress Prime Minister of India assumed the office in the year 1977.
Which of the statements given above is/are correct?
(a) 1 and 2
(b) 3 only
(c) 1 only
(d) 1 and 3
Correct Answer is Option (d)
Terms of Jawahar Lal Nehru (1947-52; 1952-57; 1957- 62; 1962-64). He died in 1964 during his 4th
Prime-ministerial term. He represented Phulpur constituency in UP. The first non-congress PM of
India was Morarji Desai (Janta Party) from 1977-1979.

Question 14: Who was the Speaker of the First Lok Sabha? [2007]
(a) Hukum Singh
(b) G.V. Mavalankar

Telegram : https://t.me/prelimbits || Personal Contact : https://t.me/EN123upsc


(c) K. M. Munshi
(d) U.N. Dhebar
Correct Answer is Option (b)
GV Mavalankar (1952-56), Hukum Singh (1962-67); K.M. Munshi and U.N. Dhebar were never the
Speakers of the Lok Sabha.

Question 15: Consider the following statements in respect of financial emergency under
Article 360 of the Constitution of India: [2007]
1. A proclamation of financial emergency issued shall cease to operate at the expiration of
two months, unless before the expiration of that period it has been approved by the
resolutions of both Houses of Parliament.
2. If any proclamation of financial emergency is in operation, it is competent for the
President of India to issue directions for the reduction of salaries and allowances of all or
any class of persons serving in connection with the affairs of the Union but excluding the
Judges of Supreme Court and the High Courts.
Which of the statements given above is/are correct?
(a) 1 only
(b) 2 only
(c) Both 1 and 2
(d) Neither 1 nor 2
Correct Answer is Option (a)
Statement 1 is correct as under Article 360, any Proclamation of Financial Emergency issued shall
cease to operate at the expiration of two months, unless before the expiration of that period it has
been approved by the resolutions Passed by both Houses of Parliament. If approved by both
Houses, then it operates for 6 months. During the proclamation of financial emergency. President
of India can issue directions for the reduction of salaries and allowances of all or any class of
persons serving in connection with the affairs of the union including the judges of supreme court
and the High warts.

Question 16: Consider the following statements: [2007]


1. The Chairman of the Committee on Public Accounts is appointed by the Speaker of the
Lok Sabha.
2. The Committee on Public Accounts comprises Members of Lok Sabha, Members of Rajya
Sabha and few eminent persons of industry and trade.
Which of the statements given above is/are correct?
(a) 1 only
(b) 2 only

Telegram : https://t.me/prelimbits || Personal Contact : https://t.me/EN123upsc


(c) Both 1 and 2
(d) Neither 1 nor 2
Correct Answer is Option (a)
Statement 2 is incorrect as Public Accounts Committee consists of 22 members: 15 from Lok Sabha
and 7 from Rajya Sabha.

Question 17: Who among the following have held the office of the Vice- President of
India? [2008]
1. Mohammad Hidayatullah
2. Fakhruddin Ali Ahmed
3. Neelam Sanjiva Reddy
4. Shankar Dayal Sharma
Select the correct answer using the code given below:
(a) 1, 2, 3 and 4
(b) 1 and 4 only
(c) 2 and 3 only
(d) 3 and 4 only
Correct Answer is Option (b)
Mohd. Hidayatullah (1979-84); Shankar Dayal Sharma (1987-92)

Question 18: Match List-I with List-II and select the correct answer using the code given
below the Lists: [2008]

(a) A-1; B-4; C-2; D-3


(b) A-2; B-3; C-1; D-4
(c) A-1; B-3; C-2; D-4
(d) A-2; B-4; C-1; D-3
Correct Answer is Option (b)
Telegram : https://t.me/prelimbits || Personal Contact : https://t.me/EN123upsc
Ashok Desai was Attorney General of India (1996-98); Nagendra Singh was President of
International Court of Justice (1985-88); AN Ray was CJI (1973-77); RK Trivedi was Chief Election
Commissioner (1982-85)

Question 19: Consider the following statements: [2008]


The Constitution of India provides that:
1. the Legislative Assembly of each State shall consist of not more than 450 members chosen
by direct election from territorial constituencies in the State
2. a person shall not be qualified to be chosen to fill a seat in the Legislative Assembly of a
State if he/she is less than 25 years of age
Which of the statements given above is/are correct?
(a) 1 only
(b) 2 only
(c) Both 1 and 2
(d) Neither 1 nor 2
Correct Answer is Option (b)
Statement 2 is correct as per provisions given under Article 173. Statement 1 is incorrect as
according to article 170, the legislative assembly of each state shall consist of not more than 500
and not less than 60 members chosen by direct election from territorial constituencies in the state.

Question 20: With reference to the Union Government, consider the following
statements: [2009]
1. The Constitution of India provides that all Cabinet Ministers shall be compulsorily be a
sitting members of Lok Sabha only.
2. The Union Cabinet Secretariat operates under the direction of the Ministry of
Parliamentary Affairs.
Which of the statements given above is/are correct?
(a) 1 only
(b) 2 only
(c) Both 1 and 2
(d) Neither 1 nor 2
Correct Answer is Option (d)
Statement 1 is incorrect as members of Rajya Sabha can become cabinet ministers. Statement 2 is
incorrect as Cabinet secretariat is under the direct charge of the PM.

Question 21: With reference to Union Government, consider the following

Telegram : https://t.me/prelimbits || Personal Contact : https://t.me/EN123upsc


statements: [2009]
1. The Ministries and Departments of the Government of India are created by the Prime
Minister on the advice of the Cabinet Secretary.
2. Each of the ministries is assigned to a Minister by the President of India on the advice of
the Prime Minister.
Which of the statements given above is/are correct?
(a) 1 only
(b) 2 only
(c) Both 1 and 2
(d) Neither 1 nor 2
Correct Answer is Option (b)
According to Article 70 the Prime Minister shall be appointed by the President and the other
Ministers shall be appointed by the President on the advice of the Prime Minister. The government
of India (Allocation of Business) Rules, 1961 is made by the President of India under Article 77 of
the constitution for the allocation of business of the government of India.
The Ministries/ Departments of the government of India are created by the president on the advice
of the prime Minister under these Rules.

Question 22: Consider the following statements: [2009]


1. The Advocate General of a State in India is appointed by the President of India upon the
recommendation of the Governor of the concerned State.
2. As provided in Civil Procedure Code, High Courts have original, appellate and advisory
jurisdiction at the State level.
Which of the statements given above is/are correct?
(a) 1 only
(b) 2 only
(c) Both 1 and 2
(d) Neither 1 nor 2
Correct Answer is Option (b)
Statement 1 is incorrect as Advocate General of the state is appointed by the governor of the
State. Statement 2 is correct. The original civil jurisdiction of the High Courts
has not altogether been abolised but retained in respect of actions of higher value.
The Original Criminal Jurisdiction of High Courts has, however, been completely taken away by the
criminal procedure code, 1973.

Question 23: Consider the following statements: [2009]

Telegram : https://t.me/prelimbits || Personal Contact : https://t.me/EN123upsc


1. Central Administrative Tribunal (CAT) was set up during the Prime Ministership of Lal
Bahadur Shastri.
2. The Members of CAT are drawn from both judicial and administrative streams.
Which of the statements given above is/are correct?
(a) 1 only
(b) 2 only
(c) Both 1 and 2
(d) Neither 1 nor 2
Correct Answer is Option (b)
Statement 1 is incorrect as CAT was set up in 1985 during the prime ministership of Rajiv Gandhi.

Question 24: With reference to Union Government, consider the following


statements: [2009]
1. Number of Ministries at the Centre on 15th August 1947 was 18.
2. Number of Ministries at the Centre at present is 36.
Which of the statements given above is/are correct?
(a) 1 only
(b) 2 only
(c) Both 1 and 2
(d) Neither 1 nor 2
Correct Answer is Option (a)
The number of ministeries at the Centre can vary based on factors such as volume of work,
importance attached to different sectors, changes of orientation of policy etc. On 15 Aug, 1947,
the number of ministries at the centre was 18.

Question 25: With reference to the Consumer Disputes Redressal at district level in India,
which one of the following statements is not correct ? [2010]
(a) A State Government can establish more than one District Forum in a district if it deems
fit.
(b) One of the members of the District Forum shall be a woman
(c) The District Forum entertains the complaints where the value of goods or services does
not exceed rupees fifty lakhs.
(d) A complaint in relation to any goods sold or any service provided may be filed with a
District Forum by the State Government as a representative of the interests
of the consumers in general.
Correct Answer is Option (c)

Telegram : https://t.me/prelimbits || Personal Contact : https://t.me/EN123upsc


The District Forum entertains the complaints where the value of goods or services does not exceed
rupees twenty lakhs.

Question 26: When the annual Union Budget is not passed by the Lok Sabha ? [2011 - I]
(a) The Budget is modified and presented again
(b) The Budget is referred to the Rajya Sabha for suggestions
(c) The Union Finance Minister is asked to resign
(d) The Prime Minister submits the resignation of Council of Ministers.
Correct Answer is Option (d)
If annual union budget is not passed by the LOK SABHA, it is tantamount to no confidence motion.
So the govt submits the resignation of his council of ministers.

Question 27: The authorization for the withdrawal of funds from the Consolidated Fund of
India must come from [2011 - I]
(a) The President of India
(b) The Parliament of India
(c) The Prime Minister of India
(d) The Union Finance Minister
Correct Answer is Option (b)
Parliament shall have power to authorise by law the withdrawal of moneys from the Consolidated
Fund of India for the purposes for which the said grants are made.

Question 28: In the Parliament of India, the purpose of an adjournment


motion is [2012 - I]
(a) to allow a discussion on a definite matter of urgent public importance
(b) to let opposition members collect information from the ministers
(c) to allow a reduction of specific amount in demand for grant
(d) to postpone the proceedings to check the inappropriate or violent behaviour on the part
of some members
Correct Answer is Option (a)
Adjournment motion:
1. It is introduced in the Parliament to draw attention of the house to a definite matter of
urgent public importance.
2. It is regarded as an extraordinary device, because it interrupts the normal business of the
house.

Telegram : https://t.me/prelimbits || Personal Contact : https://t.me/EN123upsc


3. It involves an element of censure against the government and Rajya Sabha cannot
introduce Adjournment Motion.
4. The discussion on adjournment motion should last for not less than two hours and thirty
minutes.

Question 29: Consider the following statements: [2012 - I]


1. Union Territories are not represented in the Rajya Sabha.
2. It is within the purview of the Chief Election Commissioner to adjudicate the election
disputes.
3. According to the Constitution of India, the Parliament consists of the Lok Sabha and the
Rajya Sabha only.
Which of the statements given above is/are correct?
(a) 1 only
(b) 2 and 3
(c) 1 and 3
(d) None
Correct Answer is Option (d)
None of the above statements are correct
1. Union Territories (Delhi and Pondicherry) are represented in the Rajya Sabha.
2. It is not within the purview of the Chief Election Commissioner to adjudicate election
disputes. It is the Supreme Court and High Court which look into the disputes.
3. According to the Constitution of India, the Parliament consists of the Lok Sabha, the Rajya
Sabha and the President of India.

Question 30: Regarding the office of the Lok Sabha Speaker, consider the following
statements: [2012 - I]
1. He/She holds the office during the pleasure of the President.
2. He/She need not be a member of the House at the time of his/her election but has to
become a member of the House within six months from the date of his/her election.
3. If he/she intends to resign, the letter of his/her resignation has to be addressed to the
Deputy Speaker.
Which of the statements given above is /are correct?
(a) 1 and 2 only
(b) 3 only
(c) 1, 2 and 3
(d) None
Correct Answer is Option (b)
Telegram : https://t.me/prelimbits || Personal Contact : https://t.me/EN123upsc
If the Lok Sabha Speaker wants to resign, the letter of his / her resignation has to be addressed to
the Deputy Speaker. The Speaker is elected by the Lok-Sabha from amongst its Members. Usually
the Speaker remains in office during the life of the Lok-Sabha.

Question 31: Which of the following can be said to be essentially the parts of ‘Inclusive
Governance’? [2012 - I]
1. Permitting the Non-Banking Financial Companies to do banking
2. Establishing effective District Planning Committees in all the districts
3. Increasing the government spending on public health
4. Strengthening the Mid-day Meal Scheme
Select the correct answer using the codes given below :
(a) 1 and 2 only
(b) 3 and 4 only
(c) 2, 3 and 4 only
(d) 1, 2, 3 and 4
Correct Answer is Option (c)
Inclusive governance Means that the benefit of governance Should reach the down trodden and to
the last corner of the country.
Provisions given in option 2, 3 and 4 leads to the inclusive governance.
Permitting the Non-Banking Financial companies to do banking is not directly linked to the
inclusive governance.

Question 32: In India, other than ensuring that public funds are used efficiently and for
intended purpose, what is the importance of the office of the Comptroller and Auditor
General (CAG)? [2012 - I]
1. CAG exercises exchequer control on behalf of the Parliament when the President of India
declares national emergency/financial emergency.
2. CAG reports on the execution of projects or programmes by the ministries are discussed
by the Public Accounts Committee.
3. Information from CAG reports can be used by investigating agencies to frame charges
against those who have violated the law while managing public finances.
4. While dealing with the audit and accounting of government companies, CAG has certain
judicial powers for prosecuting those who violate the law.
Which of the statements given above is/are correct?
(a) 1, 3 and 4 only
(b) 2 only

Telegram : https://t.me/prelimbits || Personal Contact : https://t.me/EN123upsc


(c) 2 and 3 only
(d) 1, 2, 3 and 4
Correct Answer is Option (c)
1. Only 2nd and 3rd are correct statements.
2. CAG reports on execution of projects or programmes by the ministries are discussed by the
Public Accounts Committee.
3. Information from CAG reports can be used by investigating agencies to press charges
against those who have violated the law while managing public finance.

Question 33: The Prime Minister of India, at the time of his/her appointment [2012 - I]
(a) need not necessarily be a member of one of the Houses of the Parliament but must
become a member of one of the Houses within six months
(b) need not necessarily be a member of one of the Houses of the Parliament but must
become a member of the Lok Sabha within six months
(c) must be a member of one of the Houses of the Parliament
(d) must be a member of the Lok Sabha
Correct Answer is Option (a)
Article 75(I) of the Indian Constitution provides that the Prime Minister shall be appointed by the
President. The Constitution permits a person to be appointed PM without his\her being a member
of either House of the Parliament at the time of appointment. However he/she has to become a
Member of either house of parliament with in Six Months from the date of his/her appointment as
prime-minister.

Question 34: A deadlock between the Lok Sabha and the Rajya Sabha calls for a joint sitting
of the Parliament during the passage of [2012 - I]
1. Ordinary Legislation
2. Money Bill
3. Constitution Amendment Bill
Select the correct answer using the codes given below :
(a) 1 only
(b) 2 and 3 only
(c) 1 and 3 only
(d) 1, 2 and 3
Correct Answer is Option (a)

Telegram : https://t.me/prelimbits || Personal Contact : https://t.me/EN123upsc


Joint sitting is an extra-ordinary Machinery provided by the constitution to resolve a deadlock
between the two houses over the passage of a bill. The provision of joint sitting is applicable to
ordinary bills of financial bills only and not to Money bills or constitutional amendment bills.

Question 35: What will follow if a Money Bill is substantially amended by the Rajya
Sabha? [2013 - I]
(a) The Lok Sabha may still proceed with the Bill, accepting or not accepting the
recommendations of the Rajya Sabha
(b) The Lok Sabha cannot consider the bill further
(c) The Lok Sabha may send the Bill to the Rajya Sabha for reconsideration
(d) The President may call a joint sitting for passing the Bill
Correct Answer is Option (a)
When a money bill returns to the Loksabha with amendments made by the Rajyasabha, it is open
to Loksabha to accept or to reject any or all of the recommendations. When the Loksabha chooses
to accept or decline the money bill with or without the recommendation, the money bill is deemed
passed in both houses.

Question 36: Which one of the following statements is correct? [2013 - I]


(a) In India, the same person cannot be appointed as Governor for two or more States at the
same time
(b) The Judges of the High Court of the States in India are appointed by the Governor of the
State just as the Judges of Supreme Court are appointed by the President
(c) No procedure has been laid down in the Constitution of India for the removal of a
Governor from his/her post
(d) In the case of a Union Territory having a legislative setup, the Chief Minister is appointed
by the Lt. Governor on the basis of majority support.
Correct Answer is Option (c)
No procedure has been laid down in the constitution of India for the removal of a governor from
his/her post.

Question 37: Consider the following statements : Attorney General of India can [2013 - I]
1. take part in the proceedings of the Lok Sabha
2. be a member of a committee of the Lok Sabha
3. speak in the Lok Sabha
4. vote in the Lok Sabha
Which of the statements given above is/are correct?
(a) 1 only

Telegram : https://t.me/prelimbits || Personal Contact : https://t.me/EN123upsc


(b) 2 and 4
(c) 1, 2 and 3
(d) 1 and 3 only
Correct Answer is Option (c)
The Attorney General of India has a post parallel to any minister in Parliament. He can take part in
the proceedings of either house. He can be a member of any committee of Parliament. He has the
right to speak in the Parliament but he has no right to vote.

Question 38: The Parliament can make any law for whole or any part of India for
implementing international treaties [2013 - I]
(a) with the consent of all the States
(b) with the consent of the majority of States
(c) with the consent of the States concerned
(d) without the consent of any State
Correct Answer is Option (d)
Parliament has exclusive power to make law with respect to any of the matters enumerated with
the Union List. According to entry no 14 in the Union List it reads- „entering into treaties and
agreements with foreign contries and implementing of treaties, agreement and convention with
foreign countries‟.

Question 39: In the context of India, which of the following principles is/are implied
institutionally in the parliamentary government? [2013 - I]
1. Members of the Cabinet are Members of the Parliament.
2. Ministers hold the office till they enjoy confidence in the Parliament.
3. Cabinet is headed by the Head of the State.
Select the correct answer using the codes given below.
(a) 1 and 2 only
(b) 3 only
(c) 2 and 3 only
(d) 1, 2 and 3
Correct Answer is Option (a)
Minister/ministers can be removed by issuing no confidence motion in the parliament. All cabinet
members are mandated by the constitution to be the member of either house of the parliament of
India. The cabinet is headed by the prime minister and is advised by the cabinet secretary who also
acts as the head of Indian Administrative service.

Telegram : https://t.me/prelimbits || Personal Contact : https://t.me/EN123upsc


Question 40: Consider the following statements: [2013 - I]
1. The Council of Ministers in the Centre shall be collectively responsible to the Parliament.
2. The Union Ministers shall hold the office during the pleasure of the President of India.
3. The Prime Minister shall communicate to the President about the proposals for
legislation.
Which of the statements given above is/are correct?
(a) 1 only
(b) 2 and 3 only
(c) 1 and 3 only
(d) 1, 2 and 3
Correct Answer is Option (b)
Article – 75 (4): The ministers Shall hold office during the pleasure of the president.
Article – 75 (5): The council of ministers shall be collectively responsible to the Lok-Sabha.
Article–78 (2): Prime minister shall furnish such information relating to the administration of the
affairs of the union and proposals for legislation on the president may call far.

Set 3
Question 1: Consider the following statements: [2013 - I]
1. The Chairman and the Deputy Chairman of the Rajya Sabha are not the members of that
House.
2. While the nominated members of the two Houses of the Parliament have no voting right
in the presidential election, they have the right to vote in the election of the Vice President.
Which of the statements given above is/are correct?
(a) 1 only
(b) 2 only
(c) Both 1 and 2
(d) Neither 1 nor 2
Correct Answer is Option (b)
The first statement is not correct because its talks about chairman as well as the deputy chairman.
The Vice- President of India is ex-officio Chairman of Rajya Sabha. Rajya Sabha also chooses from
amongst its members, a Deputy Chairman.
President: Elected members of parliament (MPs from Lok Sabha as well as Rajya Sabha). Elected
members of State legislative members, including that if NCT of Delhi and Pondicherry.
Vice President: Vice President is elected indirectly, by an electoral college consisting of members
of both houses of the parliament. The second statement is correct.

Telegram : https://t.me/prelimbits || Personal Contact : https://t.me/EN123upsc


Question 2: Consider the following statements : [2014 - I]
1. The President shall make rules for the more convenient transaction of the business of the
Government of India, and for the allocation among Ministers of the said business.
2. All executive actions of the Government of India shall be expressed to be taken in the
name of the Prime Minister.
Which of the statements given above is/are correct?
(a) 1 only
(b) 2 only
(c) Both 1 and 2
(d) Neither 1 nor 2
Correct Answer is Option (a)
(a) Clause (3) of Article 77 (“Conduct of Business of the Government of India”) of the Constitution
of India lays down as follows: “The President shall make rules for the more convenient transaction
of the business of the Government of India, and for the allocation among Ministers of the said
business.
The Constitution of India mentions that, “All executive action of the Government of India shall be
expressed to be taken in the name of the President.” Therefore, only option (a) is correct.

Question 3: The power to increase the number of judges in the Supreme Court of India is
vested in [2014 - I]
(a) the President of India
(b) the Parliament
(c) the Chief Justice of India
(d) the Law Commission
Correct Answer is Option (b)
It is the Parliament which has the power to increase the number of judges in the Supreme Court of
India. Parliament increased the number of judges from the original eight in 1950 to eleven in 1956,
fourteen in 1960, eighteen in 1978, twenty-six in 1986 and thirty-one in 2008.

Question 4: Consider the following statements regarding a No- Confidence Motion in


India: [2014 - I]
1. There is no mention of a No-Confidence Motion in the Constitution of India.
2. A Motion of No-Confidence can be introduced in the Lok Sabha only.
Which of the statements given above is/are correct?
(a) 1 only
(b) 2 only

Telegram : https://t.me/prelimbits || Personal Contact : https://t.me/EN123upsc


(c) Both 1 and 2
(d) Neither 1 nor 2
Correct Answer is Option (c)
Article 75 says that the Council of Ministers shall be collectively responsible to Loksabha. In other
words, Lok Sabha can remove them by passing a no-confidence motion. But the word “no
confidence motion” itself is not mentioned in Constitution. It comes from Rule 198 of Lok Sabha
Rules. Rajya Sabha cannot pass or remove council of ministers by passing no-confidence motion.
No confidence motion can be introduced, only in Lok Sabha.

Question 5: Which one of the following is the largest Committee of the Parliament?
(a) The Committee on Public Accounts
(b) The Committee on Estimates
(c) The Committee on Public Undertakings
(d) The Committee on Petitions
Correct Answer is Option (b)
The largest Committee is the committee of Estimates, given its 30 members

Question 6: Which of the following are the discretionary powers given to the Governor of a
State? [2014 - I]
1. Sending a report to the President of India for imposing the President’s rule
2. Appointing the Ministers
3. Reserving certain bills passed by the State Legislature for consideration of the President
of India
4. Making the rules to conduct the business of the State Government
Select the correct answer using the code given below.
(a) 1 and 2 only
(b) 1 and 3 only

Telegram : https://t.me/prelimbits || Personal Contact : https://t.me/EN123upsc


(c) 2, 3 and 4 only
(d) 1, 2, 3 and 4
Correct Answer is Option (b)
The governor has Constitutional discretion in cases of Reservation of bill for consideration of the
President and Recommendation of the imposition of President's rule. Therefore, statement 1 and 3
are definitely right. Moreover Governor only appoints those persons as ministers who are
recommended by the Chief Minister. So Governor doesn‟t have “Discretion” in appointment of the
minister. Therefore 2 is wrong.

Question 7: Which of the following is/are the function/functions of the Cabinet


Secretariat? [2014 - I]
1. Preparation of agenda for Cabinet Meetings
2. Secretarial assistance to Cabinet Committees
3. Allocation of financial resources to the Ministries
Select the correct answer using the code given below.
(a) 1 only
(b) 2 and 3 only
(c) 1 and 2 only
(d) 1, 2 and 3
Correct Answer is Option (c)
The functions of the Cabinet Secretariat includes preparation of agenda for Cabinet Meetings &
Secretarial assistance to Cabinet Committees. However Allocation of financial resources to the
Ministries as per the provisions in budget is the task of finance ministry.

Question 8: Consider the following statements : [2014 - I]


A Constitutional Government is one which
1. places effective restrictions on individual liberty in the interest of State Authority
2. places effective restrictions on the Authority of the State in the interest of individual
liberty
Which of the statements given above is/are correct?
(a) 1 only
(b) 2 only
(c) Both 1 and 2
(d) Neither 1 nor 2
Correct Answer is Option (c)
A constitutional Government needs to balance between individual liberty viz a viz State Authority

Telegram : https://t.me/prelimbits || Personal Contact : https://t.me/EN123upsc


Question 9: Consider the following statements: [2015-I]
1 The Legislative Council of a State in India can be larger in size than half of the Legislative
Assembly of that particular State
2. The Governor of a State nominates the Chairman of Legislative Council of that particular
State.
Which of the statements given above is/are correct?
(a) 1 only
(b) 2 only
(c) Both 1 and 2
(d) Neither 1 nor 2
Correct Answer is Option (d)
The maximum strength of the legislative council is fixed at one third of the total strength of the
legislative assembly and the minimum strength is fixed at 40. The chairman of the legislative
council is elected by the council itself from amongst its members.

Question 10: With reference to the Union Government, consider the following
statements: [2015-I]
1. The Department of Revenue is responsible for the preparation of Union Budget that is
presented to the Parliament.
2. No amount can be withdrawn from the Consolidated Fund of India without the
authorization from the Parliament of India.
3. All the disbursements made from Public Account also need the authorization from the
Parliament of India.
Which of the statements given above is / are correct?
(a) 1 and 2 only
(b) 2 and 3 only
(c) 2 only
(d) 1, 2 and 3
Correct Answer is Option (c)
The department of economic affairs under the ministry of finance is responsible for the
preparation of union Budget that is presented to the parliament.
 Consolidated fund of India is a fund to which all receipts are credited and all payments are
debited.
 No money can be appropriated (issued or drawn) out of the consolidated fund of India
without the authorization from the parliament of India.

Telegram : https://t.me/prelimbits || Personal Contact : https://t.me/EN123upsc


Public account of India is operated by executive action, that is, the payments from this account can
be made without parliamentary appropriation. Such payments are mostly in the nature of banking
transactions.

Question 11: There is a Parliamentary System of Government in India because the [2015-I]
(a) Lok Sabha is elected directly by the people
(b) Parliament can amend the Constitution
(c) Rajya Sabha cannot be dissolved
(d) Council of Ministers is responsible to the Lok Sabha
Correct Answer is Option (d)
The executive in a Parliamentary system is responsible 1to the legislature for all its actions. The
ministers are answerable to the parliament and responsible to the Lok Sabha. The Council of
Ministers remains in office as long as they enjoy the support and confidence of the Lok Sabha.

Question 12: Consider the following statements: [2015-I]


1. The Rajya Sabha has no power either to reject or to amend a Money Bill.
2. The Rajya Sabha cannot vote on the Demands for Grants.
3. The Rajya Sabha cannot discuss the Annual Financial Statement.
Which of the statements given above is / are correct?
(a) 1 only
(b) 1 and 2 only
(c) 2 and 3 only
(d) 1, 2 and 3
Correct Answer is Option (b)
A Money Bill cannot be introduced in Rajya Sabha. Rajya Sabha has no power either to reject or
amend a Money Bill. It can only make recommendations on the Money Bill. Rajya Sabha can
discuss the budget but cannot vote on the demands for grants ( which is the exclusive privilage of
the Lok-Sabha.

Question 13: When a bill is referred to a joint sitting of both the Houses of the Parliament, it
has to be passed by [2015-I]
(a) a simple majority of members present and voting
(b) three-fourths majority of members present and voting
(c) two-thirds majority of the Houses
(d) absolute majority of the Houses

Telegram : https://t.me/prelimbits || Personal Contact : https://t.me/EN123upsc


Correct Answer is Option (a)
In India, if a bill has been rejected by any house of the parliament and if more than six months
have elapsed, the President may summon a joint session for purpose of
passing the bill. The bill is passed by a simple majority of a joint sitting. Joint sitting is an extra-
ordinary machinery provided by the constitution to resolve a deadlock between
the two houses over the passage of a bill.
If the bill in dispute is passed by a majority of the total number of member both the houses
present and voting in the joint sitting, the bill is deemed to have been passed by both the houses.

Question 14: Consider the following statements [2015-I]


1. The Executive Power of the Union of India is vested in the Prime Minister.
2. The Prime Minister is the ex officio Chairman of the Civil Services Board.
Which of the statements given above is / are correct?
(a) 1 only
(b) 2 only
(c) Both 1 and 2
(d) Neither 1 nor 2
Correct Answer is Option (d)
The Executive powers of the Union of India is vested in the President. The Cabinet Secretary (and
not the Prime Minister) is the ex-officio head of the Civil Services Board.

Question 15: Democracy's superior virtue lies in the fact that it calls into activity [2017-I]
(a) the intelligence and character of ordinary men and women.
(b) the methods for strengthening executive leadership.
(c) a superior individual with dynamism and vision.
(d) a band of dedicated party workers.
Correct Answer is Option (a)
NEW NCERT Class 8 Civics, Chapter 3, page 32 The take-off point for a democracy is the idea of
consent, i.e. the desire, approval and participation of people. It is the decision of people that
creates a democratic government and decides about its functioning. So, since democracy requires
voters' decision making hence intelligence and character are called in. hence answer "A"

Question 16: One of the implications of equality in society is the absence of [2017-I]
(a) Privileges
(b) Restraints

Telegram : https://t.me/prelimbits || Personal Contact : https://t.me/EN123upsc


(c) Competition
(d) Ideology
Correct Answer is Option (a)
First step towards bringing about equality is of course ending the formal system of inequality and
privileges. The caste system in India prevented people from the 'lower' castes from doing anything
except manual labour. In many countries only people from some families could occupy high
positions. Attainment of equality requires that all such restrictions or privileges should be brought
to an end.

Question 17: Which of the following are not necessarily the consequences of the
proclamation of the President's rule in a State? [2017-I]
1. Dissolution of the State Legislative Assembly
2. Removal of the Council of Ministers in the State
3. Dissolution of the local bodies
Select the correct answer using the code given below:
(a) 1 and 2 only
(b) 1 and 3 only
(c) 2 and 3 only
(d) 1, 2 and 3
Correct Answer is Option (b)
MIND IT: you've to find the wrong statements herethey're the right answers.
when the President's Rule is imposed in a state, the President dismisses the state council of
ministers headed by the chief minister. The state governor, on behalf of the President, carries on
the state administration with the help of the chief secretary of the state or the advisors appointed
by the President. Meaning "2" is definitely the consequence of proclamation. Hence all options
involving "2" are wrong. Hence by elimination we are left with answer "B": 1 and 3 only.

Question 18: Which one of the following is not a feature of Indian federalism? [2017-I]
(a) There is an independent judiciary in India.
(b) Powers have been clearly divided between the Centre and the States.
(c) The federating units have been given unequal representation in the Rajya Sabha.
(d) It is the result of an agreement among the federating units.
Correct Answer is Option (d)
Indian federation is not the result of an agreement among the states unlike the American
federation. So, "D" is not the feature of Indian federalism.

Telegram : https://t.me/prelimbits || Personal Contact : https://t.me/EN123upsc


Question 19: Local self-government can be best explained as an exercise in [2017-I]
(a) Federalism
(b) Democratic decentralization
(c) Administrative delegation
(d) Direct democracy
Correct Answer is Option (b)
Balwant Rai G Mehta Committee submitted its report in November 1957 and recommended the
establishment of the scheme of 'democratic decentralisation', which ultimately came to be known
as Panchayati Raj.

Question 20: The main advantage of the parliamentary form of government is that [2017-I]
(a) the executive and legislature work independently.
(b) it provides continuity of policy and is more efficient.
(c) the executive remains responsible to the legislature.
(d) the head of the government cannot be changed without election.
Correct Answer is Option (c)
Parliamentary system is also known as Cabinet Government. It provides for collective responsibility
of the executive to the legislature. Hence answer "C".
Question 21: Out of the following statements, choose the one that brings out the principle
underlying the Cabinet form of Government: [2017-I]
(a) An arrangement for minimizing the criticism against the Government whose
responsibilities are complex and hard to carry out to the satisfaction of all.
(b) A mechanism for speeding up the activities of the Government whose responsibilities are
increasing day by day.
(c) A mechanism of parliamentary democracy for ensuring collective responsibility of the
Government to the people.
(d) A device for strengthening the hands of the head of the Government whose hold over
the people is in a state of decline.
Correct Answer is Option (c)
"Since it is not practical for all ministers to meet regularly and discuss everything, (hence) the
decisions are taken in Cabinet meetings. That is why parliamentary democracy in most countries is
often known as the Cabinet from of government." Therefore, some expert felt that cabinet from of
Government helps in work distribution and thereby speeding up efficiency so B is the right answer.
However, other experts went by the interpretation given in M. Laxmikanth's book on Indian Polity:
"Parliamentary system is also known as cabinet Government. It provides for collective responsibility

Telegram : https://t.me/prelimbits || Personal Contact : https://t.me/EN123upsc


of the executive to the legislature." So should be the answer. UPSC has kept 'C' as the official
answer.

Question 22: The Parliament of India exercises control over the functions of the Council of
Ministers through. [2017-I]
1. Adjournment motion
2. Question hour
3. Supplementary questions
Select the correct answer using the code given below:
(a) 1 only
(b) 2 and 3 only
(c) 1 and 3 only
(d) 1, 2 and 3
Correct Answer is Option (d)
The Parliament exercises control over the ministers through various devices like question hour,
discussions, adjournment motion, no confidence motion, etc. and Supplementary questions can be
asked during the question hour. Therefore, all three are correct.

Question 23: With reference to the Parliament of India, consider the following
statements: [2017-I]
1. A private member's bill is a bill presented by a Member of Parliament who is not elected
but only nominated by the President of India.
2. Recently, a private member's bill has been passed in the Parliament of India for the first
time in its history.
Which of the statements given above is/are correct?
(a) 1 only
(b) 2 only
(c) Both 1 and 2
(d) Neither 1 nor 2
Correct Answer is Option (d)
Private member's bill as a bill introduced by any member of the parliament who's not a minister.
Hence first statement is wrong.
 The Indian Express report in 2016 says only 14 private members bill have been passed since
1952. So statement 2 is also wrong.

Telegram : https://t.me/prelimbits || Personal Contact : https://t.me/EN123upsc


Question 24: For election to the Lok Sabha, a nomination paper can be filed by [2017-I]
(a) anyone residing in India.
(b) a resident of the constituency from which the election is to be contested.
(c) any citizen of India whose name appears in the electoral roll of a constituency.
(d) any citizen of India.
Correct Answer is Option (c)
For election to the Lok Sabha, a nomination paper can be filed by any citizen of India whose name
appears in the electoral roll of a constituency.

Question 25: Consider the following statements: [2017-I]


1. In the election for Lok Sabha or State Assembly, the winning candidate must get at least
50 per cent of the votes polled, to be declared elected.
2. According to the provisions laid down in the Constitution of India, in Lok Sabha, the
Speaker's post goes to the majority party and the Deputy Speaker's to the Opposition.
Which of the statements given above is/are correct?
(a) 1 only
(b) 2 only
(c) Both 1 and 2
(d) Neither 1 nor 2
Correct Answer is Option (d)
India has first past the post system wherein a candidate who wins the election may not (need to)
get majority (50%+1) votes. Statement #1 is wrong.
Upto the 10th Lok Sabha, both the Speaker and the Deputy Speaker were usually from the ruling
party. Since the 11th Lok Sabha, there has been a consensus that the Speaker comes from the
ruling party (or ruling alliance) and the post of Deputy Speaker goes to the main opposition party.
Meaning it's an 'informal consensus' among political parties, and not Constitutional provision.
Hence, statement 2 is wrong.

Question 26: Right to vote and to be elected in India is a [2017-I]


(a) Fundamental Right
(b) Natural Right
(c) Constitutional Right
(d) Legal Right
Correct Answer is Option (c)
New NCERT Class 9: Democratic Politics, Page 109: Right to vote in elections is an important
constitutional right.

Telegram : https://t.me/prelimbits || Personal Contact : https://t.me/EN123upsc


New NCERT, Std. 11, Introduction to Indian Constitution, Page 66 one of the important decisions
of the framers of India Constitution was to guarantee every adult citizen in
India, the right to vote. [Article 326]. Combining the interpretation of both textbooks, "C" is the
answer.

Question 27: Consider the following statements: [2017-I]


1. The Election Commission of India is a five-member body.
2. Union Ministry of Home Affairs decides the election schedule for the conduct of both
general elections and by-elections.
3. Election Commission resolves the disputes relating to splits/mergers of recognized
political parties.
Which of the statements given above is/are correct?
(a) 1 and 2 only
(b) 2 only
(c) 2 and 3 only
(d) 3 only
Correct Answer is Option (d)
Election Commission has three election Commissioners.
Election Commission decides the election schedule for the conduct of both general elections and
bye-elections. It also decides the disputes relating to splits/mergers of recognized political parties.
Hence answer "D" only 3.

Question 28: Consider the following statements: [2018-I]


1. In the first Lok Sabha, the single largest party in the opposition was the Swatantra Party.
2. In the Lok Sabha, a "Leader of the Opposition" was recognized for the first time in 1969.
3. In the Lok Sabha, if a party does not have a minimum of 75 members, its leader cannot be
recognized as the Leader of the Opposition.
Which of the statements given above is/are correct?
(a) 1 and 3 only
(b) 2 only
(c) 2 and 3 only
(d) 1, 2 and 3
Correct Answer is Option (b)
2 only

Telegram : https://t.me/prelimbits || Personal Contact : https://t.me/EN123upsc


 The results of the first general election did not surprise anyone…Congress party won 364 of
the 489 seats in the first Lok Sabha and finished way ahead of any other challenger. The
Communist Party of India that came next in terms of seats won only 16 seats. [NCERT
Class12: Politics in India since Independence, p30]. So, statement#1 is wrong
 In the remaining options b and c, both contain statement#2, so as such we've to accept it as
true, and no need to waste time finding proofs, Nonetheless, to satiate the curiosity: It was
Ram Subhag Singh- who became the first leader of opposition in Lok Sabha in 1969. Ref:
Indian Express
 In each House of Parliament, there is the 'Leader of the Opposition'. The leader of the
largest Opposition party having not less than one-tenth seats of the total strength of the
House is recognized as the leader of the Opposition in that House. maximum strength of
the Lok Sabha is fixed at 552 so minimum 10% doesn't translate to 75 in any case. Hence
statement#3 is wrong, by elimination, we get Answer (b)

Question 29: With reference to the Parliament of India, which of the following Parliamentary
Committees scrutinizes and reports to the House whether the powers to make regulations,
rules, sub-rules, by-laws, et(c) conferred by the Constitution or delegated by the Parliament
are being properly exercised by the Executive within the scope of such delegation ?(Pre18
Set-D) [2018-I]
(a) Committee on Government Assurances
(b) Committee on Subordinate Legislation
(c) Rules Committee
(d) Business Advisory Committee
Correct Answer is Option (b)
 Committee on government assurances- checks the assurances, promises and undertakings
given by ministers from time to time on the floor of the House and reports on the extent to
which they have been carried through. In the Lok Sabha, it consists of 15 members and in
the Rajya Sabha, it consists of 10 members. It was constituted in 1953.
 Committee on Subordinate legislation examines and reports to the House whether the
powers to make regulations, rules, sub-rules and bye-laws delegated by the Parliament or
conferred by the Constitution to the Executive are being properly exercised by it. In both
the Houses, the committee consists of 15 members. It was constituted in 1953.
 Rules committee considers the matters of procedure and conduct of business in the House
and recommends necessary amendments or additions to the rules of the House. The Lok
Sabha committee consists of 15 members including the Speaker as its ex-officio chairman.
In the Rajya Sabha, it consists of 16 members including the Chairman as its ex-officio
chairman

Telegram : https://t.me/prelimbits || Personal Contact : https://t.me/EN123upsc


 Business advisory committee regulates the programme and time table of the House. It
allocates time for the transaction of legislative and other business brought before the
House by the government. The Lok Sabha committee consists of 15 members including the
Speaker as its chairman. In the Rajya Sabha, it has 11 members including the Chairman as its
ex-officio chairman.

Question 30: Regarding Money Bill, which of the following statements is not
correct? [2018-I]
(a) A bill shall be deemed to be a Money Bill if it contains only provisions relating to
imposition, abolition, remission, alteration or regulation of any tax.
(b) A Money Bill has provisions for the custody of the Consolidated Fund of India or the
Contingency Fund of Indi(a)
(c) A Money Bill is concerned with the appropriation of moneys out of the Contingency Fund
of Indi(a)
(d) A Money Bill deals with the regulation of borrowing of money or giving of any
guarantee by the Government of Indi(a)
Correct Answer is Option (c)
 Article 110(1)(A): a Bill shall be deemed to be a Money Bill if it contains provision related
to relating to imposition, abolition, remission, alteration or regulation of any tax.
 Article 110(1)(c): the custody of the Consolidated Fund or the Contingency Fund of India,
the payment of moneys into or the withdrawal of moneys from any such Fund.
 Article 110(1)(g) any matter incidental to any of the matters specified in sub-clauses (a) to
(f) depending on how you interpret these provisions, it is difficult to outline any of the given
statement as a wrong statement. But, UPSC has kept „C‟ as the official answer.

Question 31: With reference to the election of the President of India, consider the following
statements: [2018-I]
1. The value of the vote of each MLA varies from State to State.
2. The value of the vote of MPs of the Lok Sabha is more than the value of the vote of MPs
of the Rajya Sabh(a)
Which of the statements given above is/are Correct?
(a) 1 only
(b) 2 only
(c) Both 1 and 2
(d) Neither 1 or 2
Correct Answer is Option (a)

Telegram : https://t.me/prelimbits || Personal Contact : https://t.me/EN123upsc


Statement 1- Correct: Value of 1 MLA‟s vote is based on total population of state to be divided by
the total MLAs. Hence it ought to vary from state to state. Statement 2- vaguely worded, and open
to multiple interpretation: Interpretation 2(i): While the value of an MLA‟s vote depends on the
population of the state he or she belongs to, the value of an MP‟s vote remains the same at 708 so
statement#2 is wrong. However if you interpret it as „collective‟ value then LS has more seats than
RS, the (collective) value of vote of MPs of LS > RS then #2 should be right. Accordingly, experts
were divided over answer A or C. UPSC has also kept „A‟ as the official answer.

Question 32: Consider the following statements: [2018-I]


1. The Speaker of the Legislative Assembly shall vacate his/her office if he/she ceases to be a
member of the Assembly.
2. Whenever the Legislative Assembly is dissolved, the Speaker shall vacate his/her
immediately. Which of the statements given above is/are correct?
(a) 1 only
(b) 2 only
(c) Both 1 and 2
(d) Neither 1 nor 2
Correct Answer is Option (a)
 The Speaker is elected by the assembly itself from amongst its members. Usually, the
Speaker remains in office during the life of the assembly. However, he vacates his office
earlier in any of the following three cases: 1. if he ceases to be a member of the assembly…..
So statement#1 is right.
 Constitution of India, Art 179: "…Provided further that, whenever the Assembly is dissolved,
the Speaker shall not vacate his office until immediately before the first meeting of the
Assembly after the dissolution." So, statement#2 is wrong.

Question 33: Consider the following statements: [2018-I]


1. No criminal proceedings shall be instituted against the Governor of a State any court
during his term of office.
2. The emoluments and allowances of the Governor of a State shall not be diminished during
his term of office.
Which of the statements given above is/are correct?
(a) 1 only
(b) 2 only
(c) Both 1 and 2
(d) Neither 1 nor 2
Correct Answer is Option (c)

Telegram : https://t.me/prelimbits || Personal Contact : https://t.me/EN123upsc


 Governor enjoys personal immunity from legal liability for his official acts. During his term
of office, he is immune from any criminal proceedings, even in respect of his personal acts.
He cannot be arrested or imprisone(d)
 His emoluments and allowances cannot be diminished during his term of office. Thus both
statements are right.

Question 34: If the President of India exercises his power as provided under Article 356 of
the Constitution in respect of a particular State, then [2018-I]
(a) the Assembly of the State is automatically dissolve(d)
(b) the powers of the Legislature of that State shall be exercisable by or under the authority
of the Parliament.
(c) Article 19 is suspended in that State.
(d) the President can make laws relating to that State.
Correct Answer is Option (b)
The President's Rule can be proclaimed under Article 356… He can declare that the powers of the
state legislature are to be exercised by the Parliament. So "B" is the right answer.

Question 35: The Ninth Schedule was introduced in the Constitution of India during the
prime membership of [2019-I]
(a) Jawaharlal Nehru
(b) Lal Bahadur Shastri
(c) Indira Gandhi
(d) Morarji Desai
Correct Answer is Option (a)
 "Article 31B along with the Ninth Schedule was added by the 1st Constitutional Amendment
Act of 1951 to reduce the scope of Judicial review in the matters of land reforms and
Zamindari-Abolition. Originally (in 1951), the Ninth Schedule contained only 13 acts and
regulations but at present (in 2016) their number is 282."
 So, if it was made the 50s, then it must be the Prime Minister Nehru.

Question 36: Consider the following statements: [2019-I]


1. The Parliament (Prevention of Disqualification) Act, 1959 exempts several posts from
disqualification on the grounds of 'Office of Profit'.
2. The above-mentioned Act was amended five times.
3. The term 'Office of Profit' is well-defined in the Constitution of India.

Telegram : https://t.me/prelimbits || Personal Contact : https://t.me/EN123upsc


Which of the following statements given above is/are correct?
(a) 1 and 2 only
(b) 3 only
(c) 2 and 3 only
(d) 1,2 and 3
Correct Answer is Option (a)
If 'Office of Profit' was well defined in Constitution, then there wouldn't have been controversies
related to AAP-MLAs' disqualification (2018-Jan), and Kejriwal & Co. wouldn't have been running
from pillar to post between EC and SC to prevent their disqualification. So, #3 is wrong, and by
elimination we are left with Answer A:1 and 2 only.

Question 37: With reference to the legislative Assembly of a state in India, consider the
following Statements: [2019-I]
1. The Governor makes a customary address to Members of the house at the
commencement of the first session of the year.
2. When a State Legislature does not have a rule on the particular matter, it follows the lok
Sabha rule on that matter.
(a) 1 only
(b) 2 only
(c) Both 1 and 2
(d) Neither 1 nor 2
Correct Answer is Option (c)
 Governor can address the state legislature at the commencement of the first session after
each general election and the first session of each year. So #1 is right.
 Article 208 of the Constitution: A House of the Legislature of a State may make rules for
regulating subject to the provisions of this Constitution, its procedure and the conduct of its
business. Thus, it‟s not mandated in the Constitution for the State legislature to (ALWAYS)
follow/ copy the rules of Lok Sabha. So, #2 is wrong. Thus, answer A: only 1.
 Counterview: As per IndianExpress Article in 2016-September, second statement is correct
so answer should be C: Both 1 and 2.
 Final verdict: UPSC official Answer key says “C: both statements are correct”.

Question 38: Which one of the following suggested that the Governor should be an eminent
person from outside the State and should be a detached figure without intense political
links or should not have taken part in politics in the recent past? [2019-I]
(a) First Administrative Reforms Commission (1966)
Telegram : https://t.me/prelimbits || Personal Contact : https://t.me/EN123upsc
(b) Rajamannar Committee(1969)
(c) Sarkaria Commission (1983)
(d) National Commission to Review the Working of the Constitution(2000)
Correct Answer is Option (c)
Centre-State relations:
 While selecting Governors, the Central Government should adopt the following strict
guidelines as recommended in the Sarkaria Commission report: He should be a detached
figure and not too intimately connected with the local politics of the states.

Question 39: In India, which of the following review the independent regulators in sectors
like telecommunications, insurance, electricity etc.? [2019-I]
1. Ad Hoc Committees set up by the Parliament.
2. Parliamentary Department Related Standing Committees
3. Finance Commission
4. Financial Sector Legislative Reforms Commission
5. NITI Aayog
Select the correct answer using the code given below.
(a) 1 and 2
(b) 1 , 3 and 4
(c) 3, 4 and 5
(d) 2 and 5
Correct Answer is Option (a)
 Financial Sector Legislative Reforms Commission was setup for a limited time period under
Justice B.N. Srikrishna's chairmanship. And its mandate did not include reviewing the
electricity regulator. #4 is wrong. Similar reason eliminates Finance Commission (#3) which
is setup periodically for the tax distribution between Union and States. Thus B and C are
eliminated.
 NITI Ayog is a think tank and policy advisor for the Government. Yes, it reviews
development work in the Aspirational districts and SDG implementation across India but
reviewing independent regulators is not under its purview. So, #5 is wrong. This eliminates
D. Thus by elimination, we are left with Answer A: only 1 and 2.

Question 40: In the context of polity, which one of the following would you accept as the
most appropriate definition of liberty? [2019-I]
(a) Protection against the tyranny of political rulers
(b) Absence of restraint

Telegram : https://t.me/prelimbits || Personal Contact : https://t.me/EN123upsc


(c) Opportunity to do whatever one likes
(d) Opportunity to develop oneself fully
Correct Answer is Option (d)
 Historically speaking, the term liberty was initially defined as absence of all restraints on an
individual. This is known as the negative concept of liberty. Early liberalism championed
negative liberty. John Stuart Mill, the nineteenth century English political philosopher,
described, “Restraint as an evil”. So, (d) is appropriate.
 However, after the examination, many of the coaching classes felt that answer should be “b”
based on another interpretation by D. D. Basu. But as per the official UPSC Answer key,
correct answer is “d”.

Question 41: In the context of India, which one of the following is the characteristic
appropriate for bureaucracy? [2020-I]
(a) An agency widening the scope of parliamentary democracy
(b) An agency for strengthening the structure of federalism
(c) An agency for facilitating political stability and economic growth
(d) An agency for the implementation of public policy
Correct Answer is Option (d)
Bureaucrats are appointed by the President / Governor of India for implementing the policies of
the government of the day. Other options are less suitable.

Question 42: Consider the following statements: [2020-I]


1. According to the Constitution of India, a person who is eligible to vote can be made a
minister in a state for six months even if he/she is not a member of that state.
2. According to the Representation of People Act,1951, a person convicted of a criminal
offence and sentenced to imprisonment for five years is permanently disqualified
from contesting an election even after his release from prison.
Which of the statements given above is/are correct?
(a) 1 only
(b) 2 only
(c) Both 1 and 2
(d) Neither 1 nor 2
Correct Answer is Option (a,d)
(why two answers possible)
As per the Representation of People Act, 1951, a person convicted of any offence and sentenced to
imprisonment for not less than two years shall be disqualified from the date of such conviction and
Telegram : https://t.me/prelimbits || Personal Contact : https://t.me/EN123upsc
shall continue to be disqualified for a further period of six years since his release. There is no clause
of “permanent disqualification” in the Representation of Peoples Act, 1951. Hence, 2nd statement
is incorrect. So, answer could be (a) or (d). It all depends on the validity of the first statement#1.
According to some experts, #1 is right because
 A citizen becomes eligible to vote upon attaining the age of 18.
 Whereas to become a member of the Vidhan Sabha, he must be minimum 25 years old.
 Article 164(4) of the Indian Constitution → A Minister who for any period of six consecutive
months is not a member of the Legislature of the State shall at the expiration of that period
cease to be a Minister.
 So, accordingly, some experts interpret that even 18 or 19 year old person (Who is eligible
to vote), can become minister for at least 6 months. So#1 is right.
 Other experts differ, that “since Constitution itself has not specifically said this, so
statement#1 is wrong”, so answer is “d”.
 Final judge: we‟ve to wait for the official answer key of UPSC

Question 43: Consider the following statements: [2020-I]


1. The president of India can summon a session of Parliament at such place as he/she thinks
fit.
2. The Constitution of India provides for three sessions of the Parliament in a year, but it is
not mandatory to conduct all three sessions
3. There is no minimum number of days that the Parliament is required to meet in a year.
Which of the statements given above is/are correct?
(a) 1 only
(b) 2 only
(c) 1 and 3 only
(d) 2 and 3 only
Correct Answer is Option (a,c)
 Article 85 of the Indian Constitution empowers the President to summon each house of the
Parliament at such time and place as he thinks fit. (Hence, 1st statement is correct)
 The Constitution of India does not mention the anything about the number of
Parliamentary sessions in a year. (Hence, 2nd statement is incorrect)
 There is some ambiguity in the 3rd statement.
 If the UPSC interprets it in a narrow and literal sense, then exact number of days for which
the Parliament is supposed to meet in a year is not mentioned in the Constitution and
hence, 3rd statement is correct.
Telegram : https://t.me/prelimbits || Personal Contact : https://t.me/EN123upsc
 However, if we interpret the question in a broader manner, then Article 85 of the Indian
Constitution mentions that 6 months shall not intervene the 2 sessions of the Parliament.
Interpreting this clause, it essentially means that Parliament has to meet at least for 2 days
in a year to prevent the violation of Article 85. In such a scenario, 3rd statement is incorrect.

Question 44: Rajyasabha has equal powers with Loksabha in [2020-I]


(a) The matter of creating new all India services
(b) Amending the Constitution
(c) The removal of the government
(d) Making cut motions
Correct Answer is Option (b)
 Article 312: With regards to the creation of All India Services, Rajya Sabha has special
powers such that if Rajya Sabha is passing a resolution by a majority of 2/3rd of the
members present and voting, Parliament may, by law, create one or more All India Services
in national interest. So A is wrong.
 Article 75(3) hold the government collectively responsible to the Lok Sabha. Hence, in this
regards, Lok Sabha has greater power as compared to Rajya Sabha. So C is wrong.
 Cut Motions, which are meant to reduce the amount allocated to the ministries in the
demand for grants, can be made only in Lok Sabha. Rajya Sabha can only make
recommendations with regards to the cut motion. So d is wrong. Article 368 of the Indian
Constitution, a bill to amend the Constitution can be introduced in either house of the
Parliament and both the houses have equal powers in respect to the Constitutional
Amendment Bill. So b is right.

Question 45: A constitutional government by definition is a [2020-I]


(a) government by legislature
(b) popular government
(c) Multi party government
(d) Limited government
Correct Answer is Option (d)
In India, the constitutional government is a limited government. The powers of the government are
limited by means of the Fundamental Rights enshrined in the Part-III of the Indian Constitution,
that are essentially given to us against the state actions.

Question 46: If the RBI decides to adopt an expansionist monetary policy, which of the
following would it not do ? [2020-I]

Telegram : https://t.me/prelimbits || Personal Contact : https://t.me/EN123upsc


1. Cut and optimize the Statutory Liquidity Ratio
2. Increase the Marginal Standing Facility Rate
3. Cut the Bank Rate and Repo Rate
Select the correct answer using the code given below:
(a) 1 and 2 only
(b) 2 only
(c) 1 and 3 only
(d) 1, 2 and 3
Correct Answer is Option (b)
 During the Expansionary Money / Cheap Money /Dovish Monetary Policy, RBI will try to
increase the supply of money and/or reduce the loan interest rates.
 So reducing the SLR, bank rate and repo rate will help in that regard. So, RBI may do 1 and
3. So, here question is asking, “WHAT RBI WILL NOT DO?” So, Option a, c and d are
eliminated. we are left with answer b: Only 2.
 Increasing the MSF will raise the loan interest rates, so, if RBI pursues Expansionary policy,
it‟ll not do it.

Telegram : https://t.me/prelimbits || Personal Contact : https://t.me/EN123upsc

You might also like